zeros and convergence or divergence in iterations The 2019 Stack Overflow Developer Survey Results Are InFunctional Iterations and order of convergenceImproving Newton's iteration where the derivative is near zero?Fix point iteration, rate of convergenceSome questions about variations of fixed point methodNewton-Raphson's methodPractice versus asymptotic statements about the speed of convergence.Rate of convergence of fixed-point iteration in higher dimensionsWhat is minimum number of iterations required in the bisection method to reach at the desired accuracy?Convergence of a variant of Newton's MethodAlmost sure convergence of Newton's method

What to do when moving next to a bird sanctuary with a loosely-domesticated cat?

Identify boardgame from Big movie

Did 3000BC Egyptians use meteoric iron weapons?

For what reasons would an animal species NOT cross a *horizontal* land bridge?

Is "plugging out" electronic devices an American expression?

STM32 programming and BOOT0 pin

How to answer pointed "are you quitting" questioning when I don't want them to suspect

Is there a symbol for a right arrow with a square in the middle?

How to type this arrow in math mode?

Why can Shazam fly?

FPGA - DIY Programming

Resizing object distorts it (Illustrator CC 2018)

Aging parents with no investments

Does the shape of a die affect the probability of a number being rolled?

What could be the right powersource for 15 seconds lifespan disposable giant chainsaw?

How to support a colleague who finds meetings extremely tiring?

"as much details as you can remember"

How can I autofill dates in Excel excluding Sunday?

Geography at the pixel level

How to deal with fear of taking dependencies

Can one be advised by a professor who is very far away?

Why did Acorn's A3000 have red function keys?

Output the Arecibo Message

Why isn't the circumferential light around the M87 black hole's event horizon symmetric?



zeros and convergence or divergence in iterations



The 2019 Stack Overflow Developer Survey Results Are InFunctional Iterations and order of convergenceImproving Newton's iteration where the derivative is near zero?Fix point iteration, rate of convergenceSome questions about variations of fixed point methodNewton-Raphson's methodPractice versus asymptotic statements about the speed of convergence.Rate of convergence of fixed-point iteration in higher dimensionsWhat is minimum number of iterations required in the bisection method to reach at the desired accuracy?Convergence of a variant of Newton's MethodAlmost sure convergence of Newton's method










0












$begingroup$


I'm solving some numerical analysis exercises and I still can not find a way to solve the following



Consider the function $f (x) = x^3 - x - 1$. For the equation $f (x) = 0$ answer the following:



.




  1. If we rearrange $f(x)=x(x^2-1)-1$, we arrive at the iteration:



    $x_n+1 = frac1x_n^2-1$



    When reorganizing $f (x) = (x-1)(x^2+x+1)-1$ we obtain:



    $x_n+1 = 1 + fracxx^2+x+1$



    demonstrate the equivalence of these two expressions.



  2. How do these iterations behave? (Take start point $x_0 = 1.5$ and calculate some $x_n$ values).


  3. Analyze the convergence or divergence of the previous interactions near zero $x$*.


How should I do it to get to the right way?










share|cite|improve this question









$endgroup$







  • 2




    $begingroup$
    You can with any method find some approximation for $x^*$. The execution of task 2 with 10-20 iterations should also not be too difficult, you have a computer, thus you have some scripting language or a spreadsheet calculator available. Please do that and report your results. For task 1 you need to do exactly what is demanded, show that $f(x_n)=0$ if and only if $x_n+1=x_n$, which is the equivalency.
    $endgroup$
    – LutzL
    Mar 30 at 20:10











  • $begingroup$
    Hi, thanks for your response. Regarding task 1 I have doubts. What do you mean by showing that $ f (x_n) = 0 $ and that $ x_ n + 1 = x_n $ the task is done ?, with what method could you demonstrate this? Thank you very much again
    $endgroup$
    – Fmkit
    Apr 1 at 0:36










  • $begingroup$
    You have to show that the fixed points of the iterations are exactly the roots of the polynomial. There is no method involved, just an interpretation of the factorizations that are given.
    $endgroup$
    – LutzL
    Apr 1 at 10:07















0












$begingroup$


I'm solving some numerical analysis exercises and I still can not find a way to solve the following



Consider the function $f (x) = x^3 - x - 1$. For the equation $f (x) = 0$ answer the following:



.




  1. If we rearrange $f(x)=x(x^2-1)-1$, we arrive at the iteration:



    $x_n+1 = frac1x_n^2-1$



    When reorganizing $f (x) = (x-1)(x^2+x+1)-1$ we obtain:



    $x_n+1 = 1 + fracxx^2+x+1$



    demonstrate the equivalence of these two expressions.



  2. How do these iterations behave? (Take start point $x_0 = 1.5$ and calculate some $x_n$ values).


  3. Analyze the convergence or divergence of the previous interactions near zero $x$*.


How should I do it to get to the right way?










share|cite|improve this question









$endgroup$







  • 2




    $begingroup$
    You can with any method find some approximation for $x^*$. The execution of task 2 with 10-20 iterations should also not be too difficult, you have a computer, thus you have some scripting language or a spreadsheet calculator available. Please do that and report your results. For task 1 you need to do exactly what is demanded, show that $f(x_n)=0$ if and only if $x_n+1=x_n$, which is the equivalency.
    $endgroup$
    – LutzL
    Mar 30 at 20:10











  • $begingroup$
    Hi, thanks for your response. Regarding task 1 I have doubts. What do you mean by showing that $ f (x_n) = 0 $ and that $ x_ n + 1 = x_n $ the task is done ?, with what method could you demonstrate this? Thank you very much again
    $endgroup$
    – Fmkit
    Apr 1 at 0:36










  • $begingroup$
    You have to show that the fixed points of the iterations are exactly the roots of the polynomial. There is no method involved, just an interpretation of the factorizations that are given.
    $endgroup$
    – LutzL
    Apr 1 at 10:07













0












0








0





$begingroup$


I'm solving some numerical analysis exercises and I still can not find a way to solve the following



Consider the function $f (x) = x^3 - x - 1$. For the equation $f (x) = 0$ answer the following:



.




  1. If we rearrange $f(x)=x(x^2-1)-1$, we arrive at the iteration:



    $x_n+1 = frac1x_n^2-1$



    When reorganizing $f (x) = (x-1)(x^2+x+1)-1$ we obtain:



    $x_n+1 = 1 + fracxx^2+x+1$



    demonstrate the equivalence of these two expressions.



  2. How do these iterations behave? (Take start point $x_0 = 1.5$ and calculate some $x_n$ values).


  3. Analyze the convergence or divergence of the previous interactions near zero $x$*.


How should I do it to get to the right way?










share|cite|improve this question









$endgroup$




I'm solving some numerical analysis exercises and I still can not find a way to solve the following



Consider the function $f (x) = x^3 - x - 1$. For the equation $f (x) = 0$ answer the following:



.




  1. If we rearrange $f(x)=x(x^2-1)-1$, we arrive at the iteration:



    $x_n+1 = frac1x_n^2-1$



    When reorganizing $f (x) = (x-1)(x^2+x+1)-1$ we obtain:



    $x_n+1 = 1 + fracxx^2+x+1$



    demonstrate the equivalence of these two expressions.



  2. How do these iterations behave? (Take start point $x_0 = 1.5$ and calculate some $x_n$ values).


  3. Analyze the convergence or divergence of the previous interactions near zero $x$*.


How should I do it to get to the right way?







complex-analysis numerical-methods numerical-calculus






share|cite|improve this question













share|cite|improve this question











share|cite|improve this question




share|cite|improve this question










asked Mar 30 at 16:47









FmkitFmkit

52




52







  • 2




    $begingroup$
    You can with any method find some approximation for $x^*$. The execution of task 2 with 10-20 iterations should also not be too difficult, you have a computer, thus you have some scripting language or a spreadsheet calculator available. Please do that and report your results. For task 1 you need to do exactly what is demanded, show that $f(x_n)=0$ if and only if $x_n+1=x_n$, which is the equivalency.
    $endgroup$
    – LutzL
    Mar 30 at 20:10











  • $begingroup$
    Hi, thanks for your response. Regarding task 1 I have doubts. What do you mean by showing that $ f (x_n) = 0 $ and that $ x_ n + 1 = x_n $ the task is done ?, with what method could you demonstrate this? Thank you very much again
    $endgroup$
    – Fmkit
    Apr 1 at 0:36










  • $begingroup$
    You have to show that the fixed points of the iterations are exactly the roots of the polynomial. There is no method involved, just an interpretation of the factorizations that are given.
    $endgroup$
    – LutzL
    Apr 1 at 10:07












  • 2




    $begingroup$
    You can with any method find some approximation for $x^*$. The execution of task 2 with 10-20 iterations should also not be too difficult, you have a computer, thus you have some scripting language or a spreadsheet calculator available. Please do that and report your results. For task 1 you need to do exactly what is demanded, show that $f(x_n)=0$ if and only if $x_n+1=x_n$, which is the equivalency.
    $endgroup$
    – LutzL
    Mar 30 at 20:10











  • $begingroup$
    Hi, thanks for your response. Regarding task 1 I have doubts. What do you mean by showing that $ f (x_n) = 0 $ and that $ x_ n + 1 = x_n $ the task is done ?, with what method could you demonstrate this? Thank you very much again
    $endgroup$
    – Fmkit
    Apr 1 at 0:36










  • $begingroup$
    You have to show that the fixed points of the iterations are exactly the roots of the polynomial. There is no method involved, just an interpretation of the factorizations that are given.
    $endgroup$
    – LutzL
    Apr 1 at 10:07







2




2




$begingroup$
You can with any method find some approximation for $x^*$. The execution of task 2 with 10-20 iterations should also not be too difficult, you have a computer, thus you have some scripting language or a spreadsheet calculator available. Please do that and report your results. For task 1 you need to do exactly what is demanded, show that $f(x_n)=0$ if and only if $x_n+1=x_n$, which is the equivalency.
$endgroup$
– LutzL
Mar 30 at 20:10





$begingroup$
You can with any method find some approximation for $x^*$. The execution of task 2 with 10-20 iterations should also not be too difficult, you have a computer, thus you have some scripting language or a spreadsheet calculator available. Please do that and report your results. For task 1 you need to do exactly what is demanded, show that $f(x_n)=0$ if and only if $x_n+1=x_n$, which is the equivalency.
$endgroup$
– LutzL
Mar 30 at 20:10













$begingroup$
Hi, thanks for your response. Regarding task 1 I have doubts. What do you mean by showing that $ f (x_n) = 0 $ and that $ x_ n + 1 = x_n $ the task is done ?, with what method could you demonstrate this? Thank you very much again
$endgroup$
– Fmkit
Apr 1 at 0:36




$begingroup$
Hi, thanks for your response. Regarding task 1 I have doubts. What do you mean by showing that $ f (x_n) = 0 $ and that $ x_ n + 1 = x_n $ the task is done ?, with what method could you demonstrate this? Thank you very much again
$endgroup$
– Fmkit
Apr 1 at 0:36












$begingroup$
You have to show that the fixed points of the iterations are exactly the roots of the polynomial. There is no method involved, just an interpretation of the factorizations that are given.
$endgroup$
– LutzL
Apr 1 at 10:07




$begingroup$
You have to show that the fixed points of the iterations are exactly the roots of the polynomial. There is no method involved, just an interpretation of the factorizations that are given.
$endgroup$
– LutzL
Apr 1 at 10:07










1 Answer
1






active

oldest

votes


















1












$begingroup$

Regarding 1, taking into account that the first expression is not defined for $x=1$, the expressions are only equivalent for $x ne 1$.



Regarding 3, just recall that the fixed point theorem establishes local convergence when $|g'(x^*)|<1$, and almost sure divergence when $|g'(x^*)|>1$. In the first case,
$$
|g'(x^*)| = left|dfrac-2x^*((x^*)^2-1)^2right|>1
$$

and so you conclude that the fixed point iterations will diverge unless, by pure luck, one of the iterations hit $x^*$.



In the second case, this iteration function is continuous, invariant and contractive in $[1,2]$ and therefore the iterations will converge to $x^*$ for any $x_0in [1,2]$. Moreover, considering that $|g'|leq 3/49$, we can say that
$$
|x_n-x^*| leq (3/49)^n |x_0-x^*| leq (3/49)^n.
$$






share|cite|improve this answer











$endgroup$








  • 2




    $begingroup$
    How do you get from $g'(x)=frac-2x(x^2-1)^2$ to your formula for $|g'(x^*)|$? I get $g'(x^*)=-2(x^*)^3=-2(x^*+1)$, which still proves the point but is a different formula.
    $endgroup$
    – LutzL
    Apr 1 at 10:13










  • $begingroup$
    @LutzL you are correct, I'll update the post.
    $endgroup$
    – PierreCarre
    Apr 1 at 16:44











Your Answer





StackExchange.ifUsing("editor", function ()
return StackExchange.using("mathjaxEditing", function ()
StackExchange.MarkdownEditor.creationCallbacks.add(function (editor, postfix)
StackExchange.mathjaxEditing.prepareWmdForMathJax(editor, postfix, [["$", "$"], ["\\(","\\)"]]);
);
);
, "mathjax-editing");

StackExchange.ready(function()
var channelOptions =
tags: "".split(" "),
id: "69"
;
initTagRenderer("".split(" "), "".split(" "), channelOptions);

StackExchange.using("externalEditor", function()
// Have to fire editor after snippets, if snippets enabled
if (StackExchange.settings.snippets.snippetsEnabled)
StackExchange.using("snippets", function()
createEditor();
);

else
createEditor();

);

function createEditor()
StackExchange.prepareEditor(
heartbeatType: 'answer',
autoActivateHeartbeat: false,
convertImagesToLinks: true,
noModals: true,
showLowRepImageUploadWarning: true,
reputationToPostImages: 10,
bindNavPrevention: true,
postfix: "",
imageUploader:
brandingHtml: "Powered by u003ca class="icon-imgur-white" href="https://imgur.com/"u003eu003c/au003e",
contentPolicyHtml: "User contributions licensed under u003ca href="https://creativecommons.org/licenses/by-sa/3.0/"u003ecc by-sa 3.0 with attribution requiredu003c/au003e u003ca href="https://stackoverflow.com/legal/content-policy"u003e(content policy)u003c/au003e",
allowUrls: true
,
noCode: true, onDemand: true,
discardSelector: ".discard-answer"
,immediatelyShowMarkdownHelp:true
);



);













draft saved

draft discarded


















StackExchange.ready(
function ()
StackExchange.openid.initPostLogin('.new-post-login', 'https%3a%2f%2fmath.stackexchange.com%2fquestions%2f3168509%2fzeros-and-convergence-or-divergence-in-iterations%23new-answer', 'question_page');

);

Post as a guest















Required, but never shown

























1 Answer
1






active

oldest

votes








1 Answer
1






active

oldest

votes









active

oldest

votes






active

oldest

votes









1












$begingroup$

Regarding 1, taking into account that the first expression is not defined for $x=1$, the expressions are only equivalent for $x ne 1$.



Regarding 3, just recall that the fixed point theorem establishes local convergence when $|g'(x^*)|<1$, and almost sure divergence when $|g'(x^*)|>1$. In the first case,
$$
|g'(x^*)| = left|dfrac-2x^*((x^*)^2-1)^2right|>1
$$

and so you conclude that the fixed point iterations will diverge unless, by pure luck, one of the iterations hit $x^*$.



In the second case, this iteration function is continuous, invariant and contractive in $[1,2]$ and therefore the iterations will converge to $x^*$ for any $x_0in [1,2]$. Moreover, considering that $|g'|leq 3/49$, we can say that
$$
|x_n-x^*| leq (3/49)^n |x_0-x^*| leq (3/49)^n.
$$






share|cite|improve this answer











$endgroup$








  • 2




    $begingroup$
    How do you get from $g'(x)=frac-2x(x^2-1)^2$ to your formula for $|g'(x^*)|$? I get $g'(x^*)=-2(x^*)^3=-2(x^*+1)$, which still proves the point but is a different formula.
    $endgroup$
    – LutzL
    Apr 1 at 10:13










  • $begingroup$
    @LutzL you are correct, I'll update the post.
    $endgroup$
    – PierreCarre
    Apr 1 at 16:44















1












$begingroup$

Regarding 1, taking into account that the first expression is not defined for $x=1$, the expressions are only equivalent for $x ne 1$.



Regarding 3, just recall that the fixed point theorem establishes local convergence when $|g'(x^*)|<1$, and almost sure divergence when $|g'(x^*)|>1$. In the first case,
$$
|g'(x^*)| = left|dfrac-2x^*((x^*)^2-1)^2right|>1
$$

and so you conclude that the fixed point iterations will diverge unless, by pure luck, one of the iterations hit $x^*$.



In the second case, this iteration function is continuous, invariant and contractive in $[1,2]$ and therefore the iterations will converge to $x^*$ for any $x_0in [1,2]$. Moreover, considering that $|g'|leq 3/49$, we can say that
$$
|x_n-x^*| leq (3/49)^n |x_0-x^*| leq (3/49)^n.
$$






share|cite|improve this answer











$endgroup$








  • 2




    $begingroup$
    How do you get from $g'(x)=frac-2x(x^2-1)^2$ to your formula for $|g'(x^*)|$? I get $g'(x^*)=-2(x^*)^3=-2(x^*+1)$, which still proves the point but is a different formula.
    $endgroup$
    – LutzL
    Apr 1 at 10:13










  • $begingroup$
    @LutzL you are correct, I'll update the post.
    $endgroup$
    – PierreCarre
    Apr 1 at 16:44













1












1








1





$begingroup$

Regarding 1, taking into account that the first expression is not defined for $x=1$, the expressions are only equivalent for $x ne 1$.



Regarding 3, just recall that the fixed point theorem establishes local convergence when $|g'(x^*)|<1$, and almost sure divergence when $|g'(x^*)|>1$. In the first case,
$$
|g'(x^*)| = left|dfrac-2x^*((x^*)^2-1)^2right|>1
$$

and so you conclude that the fixed point iterations will diverge unless, by pure luck, one of the iterations hit $x^*$.



In the second case, this iteration function is continuous, invariant and contractive in $[1,2]$ and therefore the iterations will converge to $x^*$ for any $x_0in [1,2]$. Moreover, considering that $|g'|leq 3/49$, we can say that
$$
|x_n-x^*| leq (3/49)^n |x_0-x^*| leq (3/49)^n.
$$






share|cite|improve this answer











$endgroup$



Regarding 1, taking into account that the first expression is not defined for $x=1$, the expressions are only equivalent for $x ne 1$.



Regarding 3, just recall that the fixed point theorem establishes local convergence when $|g'(x^*)|<1$, and almost sure divergence when $|g'(x^*)|>1$. In the first case,
$$
|g'(x^*)| = left|dfrac-2x^*((x^*)^2-1)^2right|>1
$$

and so you conclude that the fixed point iterations will diverge unless, by pure luck, one of the iterations hit $x^*$.



In the second case, this iteration function is continuous, invariant and contractive in $[1,2]$ and therefore the iterations will converge to $x^*$ for any $x_0in [1,2]$. Moreover, considering that $|g'|leq 3/49$, we can say that
$$
|x_n-x^*| leq (3/49)^n |x_0-x^*| leq (3/49)^n.
$$







share|cite|improve this answer














share|cite|improve this answer



share|cite|improve this answer








edited Apr 1 at 16:56

























answered Apr 1 at 8:59









PierreCarrePierreCarre

2,103214




2,103214







  • 2




    $begingroup$
    How do you get from $g'(x)=frac-2x(x^2-1)^2$ to your formula for $|g'(x^*)|$? I get $g'(x^*)=-2(x^*)^3=-2(x^*+1)$, which still proves the point but is a different formula.
    $endgroup$
    – LutzL
    Apr 1 at 10:13










  • $begingroup$
    @LutzL you are correct, I'll update the post.
    $endgroup$
    – PierreCarre
    Apr 1 at 16:44












  • 2




    $begingroup$
    How do you get from $g'(x)=frac-2x(x^2-1)^2$ to your formula for $|g'(x^*)|$? I get $g'(x^*)=-2(x^*)^3=-2(x^*+1)$, which still proves the point but is a different formula.
    $endgroup$
    – LutzL
    Apr 1 at 10:13










  • $begingroup$
    @LutzL you are correct, I'll update the post.
    $endgroup$
    – PierreCarre
    Apr 1 at 16:44







2




2




$begingroup$
How do you get from $g'(x)=frac-2x(x^2-1)^2$ to your formula for $|g'(x^*)|$? I get $g'(x^*)=-2(x^*)^3=-2(x^*+1)$, which still proves the point but is a different formula.
$endgroup$
– LutzL
Apr 1 at 10:13




$begingroup$
How do you get from $g'(x)=frac-2x(x^2-1)^2$ to your formula for $|g'(x^*)|$? I get $g'(x^*)=-2(x^*)^3=-2(x^*+1)$, which still proves the point but is a different formula.
$endgroup$
– LutzL
Apr 1 at 10:13












$begingroup$
@LutzL you are correct, I'll update the post.
$endgroup$
– PierreCarre
Apr 1 at 16:44




$begingroup$
@LutzL you are correct, I'll update the post.
$endgroup$
– PierreCarre
Apr 1 at 16:44

















draft saved

draft discarded
















































Thanks for contributing an answer to Mathematics Stack Exchange!


  • Please be sure to answer the question. Provide details and share your research!

But avoid


  • Asking for help, clarification, or responding to other answers.

  • Making statements based on opinion; back them up with references or personal experience.

Use MathJax to format equations. MathJax reference.


To learn more, see our tips on writing great answers.




draft saved


draft discarded














StackExchange.ready(
function ()
StackExchange.openid.initPostLogin('.new-post-login', 'https%3a%2f%2fmath.stackexchange.com%2fquestions%2f3168509%2fzeros-and-convergence-or-divergence-in-iterations%23new-answer', 'question_page');

);

Post as a guest















Required, but never shown





















































Required, but never shown














Required, but never shown












Required, but never shown







Required, but never shown

































Required, but never shown














Required, but never shown












Required, but never shown







Required, but never shown







Popular posts from this blog

Triangular numbers and gcdProving sum of a set is $0 pmod n$ if $n$ is odd, or $fracn2 pmod n$ if $n$ is even?Is greatest common divisor of two numbers really their smallest linear combination?GCD, LCM RelationshipProve a set of nonnegative integers with greatest common divisor 1 and closed under addition has all but finite many nonnegative integers.all pairs of a and b in an equation containing gcdTriangular Numbers Modulo $k$ - Hit All Values?Understanding the Existence and Uniqueness of the GCDGCD and LCM with logical symbolsThe greatest common divisor of two positive integers less than 100 is equal to 3. Their least common multiple is twelve times one of the integers.Suppose that for all integers $x$, $x|a$ and $x|b$ if and only if $x|c$. Then $c = gcd(a,b)$Which is the gcd of 2 numbers which are multiplied and the result is 600000?

Ingelân Ynhâld Etymology | Geografy | Skiednis | Polityk en bestjoer | Ekonomy | Demografy | Kultuer | Klimaat | Sjoch ek | Keppelings om utens | Boarnen, noaten en referinsjes Navigaasjemenuwww.gov.ukOffisjele webside fan it regear fan it Feriene KeninkrykOffisjele webside fan it Britske FerkearsburoNederlânsktalige ynformaasje fan it Britske FerkearsburoOffisjele webside fan English Heritage, de organisaasje dy't him ynset foar it behâld fan it Ingelske kultuergoedYnwennertallen fan alle Britske stêden út 'e folkstelling fan 2011Notes en References, op dizze sideEngland

Boston (Lincolnshire) Stedsbyld | Berne yn Boston | NavigaasjemenuBoston Borough CouncilBoston, Lincolnshire